[MIX] Świętokrzyskie warsztaty, wrzesień 2007

Zadania z kółek matematycznych lub obozów przygotowujących do OM. Problemy z minionych olimpiad i konkursów matematycznych.
Regulamin forum
Wszystkie tematy znajdujące się w tym dziale powinny być tagowane tj. posiadać przedrostek postaci [Nierówności], [Planimetria], itp.. Temat może posiadać wiele różnych tagów. Nazwa tematu nie może składać się z samych tagów.
Awatar użytkownika
Sylwek
Użytkownik
Użytkownik
Posty: 2716
Rejestracja: 21 maja 2007, o 14:24
Płeć: Mężczyzna
Lokalizacja: Warszawa
Podziękował: 160 razy
Pomógł: 657 razy

[MIX] Świętokrzyskie warsztaty, wrzesień 2007

Post autor: Sylwek »

Już po raz czwarty odbyły się warsztaty matematyczne organizowane przez Naukowe Koło Matematyczne Funkcjonał. Finaliści OM z naszego województwa prowadzili wykłady o wielomianach, inwersji, przystawaniu trójkątów, najmniejszej odległości, znajdowanie wzoru ogólnego ciągu na podstawie wzoru rekurencyjnego, teorii grup w odniesieniu do Twierdzenia Eulera o liczbach względnie pierwszych, jedna osoba dostała również godzinę na szybki wykład o rachunku całkowym i różniczkowym. Osobiście pojawiłem się tam pierwszy raz, poziom zadanek zróżnicowany, raczej nie olimpijski, ale w sumie sporo się nauczyłem. Oto zadanka:



* 1. dzień, spotkanie przedpołudniowe:

1. Uprość wyrażenia: a) \(\displaystyle{ \frac{x^8+x^6+x^4+x^2+1}{x^4+x^3+x^2+x+1}}\) b) \(\displaystyle{ \frac{x^{4n}+x^{4n-2}+ \ldots + x^2+1}{x^{2n}+x^{2n-1}+ \ldots + x+1}}\).

2. W rebusie KAR+KRA=RAK rozszyfrować, jaką liczbą jest RAK (każda litera odpowiada innej cyfrze).

3. Wykazać, że liczba \(\displaystyle{ 2^{10}+5^{12}}\) jest złożona.

4. Znaleźć wszystkie funkcje rzeczywiste przyjmujące wartości nieujemne spełniające dla wszystkich liczb x, y warunek \(\displaystyle{ f(x+y) \geq f(x) + f(y)}\).

5. Rozwiązać w liczbach całkowitych równanie \(\displaystyle{ 9x^2-39x+40=y^2}\).

6. Znaleźć wszystkie liczby całkowite \(\displaystyle{ x \neq 3}\), takie że \(\displaystyle{ x-3}\) dzieli \(\displaystyle{ x^3-3}\).

7. Udowodnić, że jeżeli a,b,c są długościami boków trójkąta to \(\displaystyle{ |\frac{a-b}{a+b}+\frac{b-c}{b+c}+\frac{c-a}{c+a}| \leq \frac{1}{8}}\)

8. Ponumerować wierzchołki sześcianu liczbami ze zbioru {1, 2, 3, ... , 9} tak, aby sumy numerów wierzchołków należących do każdej ściany były równe i niepodzielne przez liczbę nie wziętą do numeracji.




* 1. dzień, spotkanie popołudniowe:

1. Czy istnieją różne liczby pierwsze \(\displaystyle{ p_{1}, p_{2}, \ldots , p_{n} \ (n \geq 2)}\) takie, że liczba \(\displaystyle{ \frac{1}{\frac{1}{p_{1}}+\frac{1}{p_{2}}+ \ldots + \frac{1}{p_{n}}}}\) jest całkowita?

2. Udowodnić, że jeżeli \(\displaystyle{ a+b+c=0}\) i \(\displaystyle{ a^3+b^3+c^3=0}\), to dla każdego naturalnego n \(\displaystyle{ a^{2n+1}+b^{2n+1}+c^{2n+1}=0}\).

3. Pewną liczbę łuków danego okręgu pomalowano na czarno, przy czym suma długości tych łuków jest mniejsza od połowy obwodu tego okręgu. Udowodnić, że istnieje średnica tego okręgu o nie pomalowanych końcach.

4. Na boku AB i na przekątnej AC kwadratu ABCD obrano odpowiednio takie punkty P i Q, że AP:PB=3:2 i AQ:QC=4:1 . Udowodnić, że kąt PQD jest prosty.

5. Znaleźć wszystkie funkcje \(\displaystyle{ f: \mathbb{R} \rightarrow \mathbb{R}}\), które dla dowolnych liczb rzeczywistych x i y spełniają równanie \(\displaystyle{ f(x+y)-f(x-y)=4xy}\).

6. W dwóch naczyniach znajduje się odpowiednio m i n litrów roztworu o różnych stężeniach. Z obu naczyń odlano taką samą ilość roztworu. Następnie roztwór odlany z naczynia m-litrowego wlano do naczynia n-litrowego i podobnie odlany roztwór z naczynia n-litrowego wlano do naczynia m-litrowego. Okazało się, że stężenia obu roztworów wyrównały się. Ile litrów roztworu odlano z każdego naczynia?

7. Wykazać, że \(\displaystyle{ \cos \alpha + \sin \alpha \leq \sqrt{2}}\).




* 2. dzień, spotkanie przedpołudniowe:

1. Znaleźć wszystkie liczby pierwsze p, dla których liczby p+10 i p+20 również są pierwsze.

2. Oblicz wartość wyrażenia \(\displaystyle{ a^2+b^2+c^2-abc}\) dla \(\displaystyle{ a=99\frac{1}{99}, \ b=101\frac{1}{101}, \ c=9999\frac{1}{9999}}\).

3. Udowodnij, że jeżeli n jest liczbą naturalną nieparzystą i większą od 1, to istnieje taka liczba naturalna m mniejsza od n, dla której \(\displaystyle{ n|2^m-1}\).

4. Udowodnić, że dla nieparzystych n mamy \(\displaystyle{ n|2^{n!}-1}\).

5. Dowieść, że równanie \(\displaystyle{ x^2-2y^2+8z=3}\) nie ma rozwiązań w liczbach całkowitych.

6. Ile jest liczb trzycyfrowych niepodzielnych przez żadną z liczb: 3, 5, 7?

7. Usunąć niewymierność z mianownika: \(\displaystyle{ \frac{1}{1+\sqrt{3}-\sqrt[3]{2}}}\).

8. Obliczyć sumę \(\displaystyle{ \frac{1 \cdot 2!}{2}+\frac{2 \cdot 3!}{2^2}+\frac{3 \cdot 4!}{2^3}+\ldots+\frac{n \cdot (n+1)!}{2^n}}\).

9. Pokazać, że \(\displaystyle{ a=\sqrt{2}+\sqrt{3}}\) jest liczbą niewymierną.

10. Pokazać, że jeżeli \(\displaystyle{ 7|2a+3b}\) oraz \(\displaystyle{ 7|5a-b}\), to \(\displaystyle{ 7|a \ \wedge \ 7|b}\)




* 2. dzień, spotkanie popołudniowe:

1. Udowodnić, że jeżeli \(\displaystyle{ \alpha, \ \beta, \ \gamma}\) są kątami między przekątnymi ścian prostopadłościanu wychodzącymi z jednego wierzchołka, to \(\displaystyle{ \alpha+\beta+\gamma=180^{\circ}}\).

2. Odcinki AB i CD są styczne do dwóch okręgów, do pierwszego w punktach A i C, do drugiego w punktach B i D. Udowodnić, że rzuty odcinków AC i BD na prostą łączącą środki tych okręgów są równej długości.

3. Wyznacz wszystkie pary (p,q) liczb pierwszych, dla których równanie \(\displaystyle{ x^4-px^3+q=0}\) ma pierwiastek całkowity.

4. Naszkicuj zbiór wszystkich punktów P(x,y) płaszczyzny takich, że \(\displaystyle{ x^2+x=y^2+y}\).

5. Wykres pewnej funkcji liniowej przechodzi przez dwa punkty o obu współrzędnych całkowitych. Wykazać, że do wykresu tej funkcji należy nieskończenie wiele punktów o obu współrzędnych całkowitych.

6. Rozwiązać równanie \(\displaystyle{ [x]=\frac{x+1}{2}}\).

7. Wykazać, że suma odległości punktu P leżącego wewnątrz trójkąta równobocznego od boków nie zależy od położenia punktu P.

8. Znaleźć wszystkie wielomiany P(x), dla których zachodzi następująca tożsamość: \(\displaystyle{ (x-26) \cdot P(x)=x \cdot P(x-1)}\).




* 3. dzień, spotkanie przedpołudniowe:

1. Pokazać, że \(\displaystyle{ m|n}\) wtedy i tylko wtedy, gdy \(\displaystyle{ 2^m-1|2^n-1}\).

2. Znaleźć wielomian o współczynnikach całkowitych, którego pierwiastkiem jest liczba \(\displaystyle{ a=\sqrt{2}+\sqrt{3}}\).

3. Pokazać, że \(\displaystyle{ 1985|1983!!+1984!!}\)

4. W pola nieskończonej szachownicy wpisano liczby naturalne w ten sposób, że każda liczba w polu jest średnią arytmetyczną ośmiu liczb z nią sąsiadujących. Pokazać, że liczba 100 pojawia się na szachownicy nieskończenie wiele razy albo nie pojawia się wcale.

5. W turnieju szachowym rozgrywanym systemem "każdy z każdym" brało udział czterech zawodników. Za zwycięstwo szachista otrzymuje 1 pkt, za remis 0,5 pkt, a za przegraną 0 pkt. Po zakończeniu turnieju okazało się, że każdy z zawodników otrzymał inną liczbę punktów. Wykazać, że zawodnik, który zajął ostatnie miejsce, nie wygrał żadnej partii.

6. Wyznaczyć wzór na objętość ostrosłupa ściętego mając dane pole podstawy dolnej S1, pole podstawy górnej S2 oraz wysokość h.

7. Dany jest 2n-kąt foremny \(\displaystyle{ A_{1}A_{2} \ldots A_{2n}}\). P jest dowolnym punktem okręgu opisanego na tym wielokącie. Pokazać, że suma \(\displaystyle{ |PA_{1}|^2+|PA_{2}|^2+\ldots+|PA_{2n}|^2}\) nie zależy od wyboru punktu P.




* 3. dzień, spotkanie popołudniowe:

1. Podać przykład wielomianu, który dla m różnych liczb naturalnych przyjmuje wartości będące różnymi liczbami pierwszymi.

2. Liczby p i 8p-1 są pierwsze. Wykazać, że liczba 8p+1 jest złożona.

3. Wyznaczyć wszystkie rozwiązania całkowitoliczbowe równania 3x+7y=1

4. Iloma zerami kończy się rozwinięcie dziesiętne liczby \(\displaystyle{ 100!}\)?

5. Załóżmy, że a jest k-krotnym pierwiastkiem wielomianu P i l-krotnym pierwiastkiem wielomianu Q, przy czym \(\displaystyle{ k \le l}\). Pokazać, że a jest co najmniej k-krotnym pierwiastkiem wielomianu \(\displaystyle{ P(x)-Q(x)}\).

6. Pokazać, że wielomian \(\displaystyle{ x^{100}y^{100}+1}\) nie jest iloczynem wielomianów postaci f(x) i g(y).

7. Pokazać, że proste łączące środki przeciwległych krawędzi czworościanu przecinają się w jednym punkcie.

8. Niech ABC będzie takim trójkątem, że \(\displaystyle{ \angle ABC=90^{\circ}}\). Punkty D i E wybrano na prostych BC i AB w taki sposób, że AB=BD, BC=BE i punkty leżą na odpowiednich prostych w kolejności A,B,E i C,B,D. Pokazać, że wysokość poprowadzona z wierzchołka B dzieli odcinek DE na równe części.




* 4. dzień, mecz matematyczny:


1. Pokazać, że środkowe czworościanu przecinają się w jednym punkcie.

2. Udowodnić, że liczba rzeczywista ma okresowe rozwinięcie dziesiętne wtedy i tylko wtedy gdy jest wymierna.

3. Na płaszczyźnie danych jest 2n punktów, z których żadne trzy nie leżą na jednej prostej. Udowodnić, że można połączyć punkty za pomocą n odcinków, tak aby żadne dwa odcinki nie miały punktu wspólnego.

4. Rozstrzygnąć, czy istnieje trójkąt równoboczny o wierzchołkach w punktach kratowych.

5. Rozważmy domino o polach ponumerowanych liczbami 0, 1, ..., n (np. dla n=6 mamy 28 klocków). Rozstrzygnąć dla jakich n można stosując reguły domina ułożyć pętlę.

6. Wyznacz wszystkie liczby pierwsze p i q takie, że: \(\displaystyle{ p^3-q^5=(p+q)^2}\).

7. Przy okrągłym, obrotowym stole w restauracji usiedli goście, przy czym każdy pomylił miejsce i w rezultacie żaden nie dostał tego co zamawiał na obiad. Pokazać, że można tak obrócić stół, aby co najmniej dwie osoby siedziały naprzeciwko swojego nakrycia.

8. W kwadracie 1x1 znajduje się 101 punktów. Pokazać, że na pewnych trzech można opisać trójkąt o polu nie większym niż 0,01.

9. Czy na płaszczyźnie może istnieć układ różnych punktów A1, A2, ..., A2007 i prosta, która przecina wnętrze każdego z odcinków \(\displaystyle{ \overline{A_{1}A_{2}}, \ \overline{A_{2}A_{3}}, \ \ldots, \ \overline{A_{2006}A_{2007}}, \ \overline{A_{2007}A_{1}}}\)

10. Płaszczyznę podzielono za pomocą prostych na pewną liczbę obszarów. Pokazać, że można pokolorować obszary na czarno i biało, tak aby sąsiednie obszary nie były tego samego koloru.

11. Trzy punkty leżące na trzech bokach trójkąta nazwiemy trójką Cevy, jeżeli proste łączące te punkty z wierzchołkami przecinają się w jednym punkcie. Przypuśćmy, że wykreśliliśmy okrąg przez punkty tworzące trójkę Cevy otrzymując trzy nowe punkty na bokach trójkąta. Udowodnij, że nowe punkt także tworzą trójkę Cevy.




* 5. dzień, spotkanie przedpołudniowe:

1. Niech x, y będą liczbami nieujemnymi takimi, że x≥y. Wykaż, że \(\displaystyle{ x^4+y^4 \geq 2xy^3}\).

2. Na ile sposobów można posadzić pewną ilość osób na n krzesłach (n≥1) w ten sposób, aby pomiędzy każdymi dwiema osobami było co najmniej jedno krzesło wolne? Krzesła stoją w rzędzie jedno obok drugiego.
UWAGA: Przyjmujemy puste krzesła jako jeden ze sposobów.

3. Prędkość ruchu pewnego punktu materialnego opisana jest zależnością od czasu v=at+v0. Przyjmujemy, że zarówno prędkość początkowa v0 jak i przyspieszenie a są dodatnie. Oblicz drogę, jaką pokona punkt materialny w czasie od t1 do t2 (0≤t1≤t2). Jaką drogę pokona w czasie od 0 do t?
UWAGA. Droga ta opisana jest wzorem \(\displaystyle{ s=\int\limits_{t_{1}}^{t_{2}} v(t) \ dt}\), czyli jest to pole powierzchni pod wykresem prędkości ograniczone od dołu osią czasu oraz prostymi t=t1, t=t2.

4. Rozłóż na czynniki wielomian \(\displaystyle{ x^4+x^2+1}\).

5. Rozwiąż układ równań:
\(\displaystyle{ \begin{cases}x^4+x^2y^2+y^4=21 \\ x^2-xy+y^2=3 \end{cases}}\)

6. Dany jest trójkąt ABC taki, że spodek wysokości z wierzchołka C leży we wnętrzu podstawy. Na bokach AC i BC zbudowano dwa skierowane na zewnątrz trójkąty prostokątne BCD i ACE takie, że kąty EAC i DBC są proste, zaś kąty ACE i BCD są równe. Uzasadnij, że proste BE, AD oraz wysokość poprowadzona z wierzchołka C przecinają się w jednym punkcie.

7. Danych jest pięć różnych liczb naturalnych dodatnich. Pokazać, że można wybrać trzy z nich tak, aby suma była podzielna przez trzy.





Powodzenia!
P.S. Kolorem czerwonym będę oznaczał numery zadań, które zostały (zostaną) poniżej rozwiązane.
Ostatnio zmieniony 1 paź 2008, o 18:24 przez Sylwek, łącznie zmieniany 4 razy.
Piotr Rutkowski
Użytkownik
Użytkownik
Posty: 2234
Rejestracja: 26 paź 2006, o 18:08
Płeć: Mężczyzna
Lokalizacja: Warszawa
Podziękował: 22 razy
Pomógł: 390 razy

[MIX] Świętokrzyskie warsztaty, wrzesień 2007

Post autor: Piotr Rutkowski »

Heh, za sekundę idę do szkoły, więc tylko przeglądnąłem zadanka i na razie wymyśliłem.
1)Tu po prostu zastosować wzór na sumę ciągu geometrycznego w liczniku i mianowniku.
W punkcie b również.

1)spotkanie popołudniowe:
jesli naszą liczbę całkowitą oznaczymy jako m to dostaniemy:
\(\displaystyle{ \frac{1}{p_{1}}+\frac{1}{p_{2}}+...+\frac{1}{p_{n}}=\frac{1}{m}}\)
Niech \(\displaystyle{ q=p_{1}p_{2}...p_{n}}\) mamy:
\(\displaystyle{ \frac{q}{p_{1}}+\frac{q}{p_{2}}+...+\frac{q}{p_{n}}=\frac{q}{m}}\) lewa i prawa strona są całkowite wobec tego \(\displaystyle{ m|q}\), a więc m=1 lub jest iloczynem jakichś naszych liczb pierwszych. Niestety nie mam czasu dopisać teraz reszty dowodu, dokończę to jakoś weczorem.

OK, jestem już na forum, dokończę to zadanie:
Załóżmy zatem, że m=1
wtedy liczby:\(\displaystyle{ \frac{q}{p_{2}},\frac{q}{p_{3}}...,\frac{q}{p_{n}},\frac{q}{m}=q}\), są podzielne przez \(\displaystyle{ p_{1}}\), a liczba \(\displaystyle{ \frac{q}{p_{1}}=p_{2}p_{3}...p_{n}}\), nie jest, co prowadzi nas do sprzeczności

Załóżmy bez zmniejszania ogólności rozważań, że \(\displaystyle{ m=p_{1}p_{2}...p_{k}}\)
Z równania:\(\displaystyle{ \frac{q}{p_{1}}+\frac{q}{p_{2}}+...+\frac{q}{p_{n}}=\frac{q}{m}}\) mamy, że \(\displaystyle{ \frac{q}{p_{1}}m=p_{1}p_{2}...p_{k}}\), co jest niemożliwe. Otrzymana sprzeczność dowodzi, że teza podana w zadaniu jest fałszywa
Ostatnio zmieniony 20 wrz 2007, o 15:44 przez Piotr Rutkowski, łącznie zmieniany 1 raz.
Awatar użytkownika
mol_ksiazkowy
Użytkownik
Użytkownik
Posty: 11265
Rejestracja: 9 maja 2006, o 12:35
Płeć: Mężczyzna
Lokalizacja: Kraków
Podziękował: 3143 razy
Pomógł: 747 razy

[MIX] Świętokrzyskie warsztaty, wrzesień 2007

Post autor: mol_ksiazkowy »

2. Znaleźć wielomian o współczynnikach całkowitych, którego pierwiastkiem jest liczba \(\displaystyle{ a=\sqrt{2}+\sqrt{3}}\).

\(\displaystyle{ w(x)= (x- (\sqrt{2}+\sqrt{3}))(x+ (\sqrt{2}+\sqrt{3})(x^2-5-2\sqrt{6})=x^4-10x^2+1}\).
Awatar użytkownika
Tristan
Użytkownik
Użytkownik
Posty: 2353
Rejestracja: 24 kwie 2005, o 14:28
Płeć: Mężczyzna
Podziękował: 27 razy
Pomógł: 557 razy

[MIX] Świętokrzyskie warsztaty, wrzesień 2007

Post autor: Tristan »

Ad 1.pe.3:
Zauważmy, że\(\displaystyle{ 2^{10} +5^{12}=(2^5)^2 +(5^6)^2=(2^5)^2 + 2 \cdot 2^5 \cdot 5^5 + (5^6)^2 -2 \cdot 2^5 \cdot 5^5 = (2^5 + 5^6)^2 - 2^6 \cdot 5^5=(2^5 + 5^6)^2 - (10^3)^2= (2^5+5^6 - 10^3)(2^5 + 5^6 +10^3}\)
A ponieważ \(\displaystyle{ 5^3 - 2^3 >1}\), więc \(\displaystyle{ 5^3 ( 5^3 - 2^3) +2^5=5^6 - 10^3 +2^5>1}\), więc rzeczywiście \(\displaystyle{ 2^{10} +5^{12}}\) jest liczbą złożoną.

Ad 1.po.7:
Korzystając z takiej wersji nierówności Schwarza \(\displaystyle{ (a_{1}^2 + a_{2}^2 )( b_{1}^2 + b_{2}^2) \geq (a_{1}b_{1}+a_{2}b_{2})^2}\) ( kto nie wierzy, że zachodzi - niech wszystko wymnoży, a ładnie się poskracają niektóre wyrażenia) mamy \(\displaystyle{ (\cos x+ \sin x)^2=(1 \cos x + 1 \sin x)^2 \leq ( 1^2 +1^2)( \cos^2 x + \sin^2 x)=2 1=2}\), więc \(\displaystyle{ \cos x+ \sin x \leq |\cos x+ \sin x| \leq \sqrt{2}}\).

Ad 2.po.3:
Korzystając z twierdzenia o wymiernych pierwiastkach wielomianu o współczynnikach całkowitych dostajemy, że jedynymi pierwiastkami całkowymi tego wielomianu mogą być q i -q. Jednak w tym drugim przypadku otrzymalibyśmy, że \(\displaystyle{ q^4 +pq^3+q=0}\) - a to jest sprzeczność, bo po lewej stronie mamy sumę liczb dodatnich. Rozważamy więc przypadek, gdy \(\displaystyle{ x=q}\). Mamy wtedy równanie \(\displaystyle{ q^4 - pq^3+q=0}\). Oczywiście \(\displaystyle{ q \ne 0}\), więc możemy podzielić to równanie przez q otrzymując \(\displaystyle{ q^3 +1= pq^2}\). Zauważmy, że \(\displaystyle{ q \ne 2\land p\ne 2}\). Czyli \(\displaystyle{ q,p \geq 3}\). Jednak lewa strona rónania\(\displaystyle{ q^3 +1=(q+1)(q^2 - q+1)}\) jest liczbą parzystą ze względu na czynnik \(\displaystyle{ q+1}\), a prawa \(\displaystyle{ pq^2}\) nieparzystą - sprzeczność. Otrzymujemy więc, że nie istnieje taka para liczb pierwszych (p,q) spełniająca warunki zadania.

Ad 5.pe.1:
Z nierówności między średnią arytmetyczną i geometryczną mamy, że \(\displaystyle{ x^4 + y^4 \geq 2 x^2 y^2}\). A ponieważ \(\displaystyle{ 2x^2 y^2 -2xy^3= 2xy^2 (x-y) \geq 0}\), więc dana nierówność jest prawdziwa.
Ostatnio zmieniony 20 wrz 2007, o 15:53 przez Tristan, łącznie zmieniany 1 raz.
Piotr Rutkowski
Użytkownik
Użytkownik
Posty: 2234
Rejestracja: 26 paź 2006, o 18:08
Płeć: Mężczyzna
Lokalizacja: Warszawa
Podziękował: 22 razy
Pomógł: 390 razy

[MIX] Świętokrzyskie warsztaty, wrzesień 2007

Post autor: Piotr Rutkowski »

Dzień piąty zadanie 1:
Ze średnich mamy \(\displaystyle{ x^{4}+y^{4}\geq 2\sqrt{x^{4}y^{4}}=2x^{2}y^{2}}\)
udowodnijmy, że \(\displaystyle{ x^{2}y^{2}\geq xy^{3}}\)
dzieląc przez \(\displaystyle{ xy^{2}}\) otrzymujemy \(\displaystyle{ x\geq y}\) co jest prawdą (lub w tym miejscu stosujemy ciągi jednomonotoniczne )

Dzień 2 zadanie 10:
\(\displaystyle{ 2a+3b \equiv 0 (mod7)}\)
\(\displaystyle{ 3*(5a-b)\equiv 0 (mod7}\) dodając stronami:
\(\displaystyle{ 17a\equiv 0 (mod 7) \Rightarrow a\equiv 0 (mod7)}\)

Dzień 2 zadanie 9
\(\displaystyle{ a \in Q \Rightarrow a^{2}\in Q}\)

Dzień 1 zadanie2:
skorzystamy z tożsamości \(\displaystyle{ \frac{1}{2}(a+b+c)((a-b)^{2}+(b-c)^{2}+(c-a)^{2})=a^{3}+b^{3}+c^{3}-3abc}\)
z tego i z danych w zadaniu wynika, że \(\displaystyle{ 3abc=0}\), a więc co najmniej jedna z liczb a,b,c jest równa zero (lub też wszystkie). Jeśli tylko jedna z nich jest równa zero, to pozostałe dwie są liczbami przeciwnymi c.n.d.
Awatar użytkownika
max
Użytkownik
Użytkownik
Posty: 3306
Rejestracja: 10 gru 2005, o 17:48
Płeć: Mężczyzna
Lokalizacja: Lebendigentanz
Podziękował: 37 razy
Pomógł: 778 razy

[MIX] Świętokrzyskie warsztaty, wrzesień 2007

Post autor: max »

Dz 1 przed 4.
Indukcyjnie dla dowolnych \(\displaystyle{ n\in \mathbb{N},\ x\in\mathbb{R}}\):
\(\displaystyle{ f(nx) \geqslant nf(x)}\)
Stąd:
\(\displaystyle{ f(0) = f(nx + (-nx)) \geqslant f(nx) + f(-nx) \geqslant nf(x) + nf(-x)}\)
czyli wyrażenie \(\displaystyle{ n\big(f(x) + f(-x)\big), \ x\in \mathbb{R}, n\in \mathbb{N}}\) powinno być ograniczone (od góry), a będzie tak wtw, gdy \(\displaystyle{ f}\) będzie nieparzysta, ale ponieważ \(\displaystyle{ f}\) może przyjmować jedynie wartości nieujemne, to \(\displaystyle{ \forall x\in \mathbb{R} \ \big(f(x) = 0\big)}\)

Dz 1 po 5.
Podkładając \(\displaystyle{ x = y = \frac{u + v}{2}}\) dostajemy:
\(\displaystyle{ f(u + v) = (u + v)^{2} + f(0)}\)
zatem dla każdego \(\displaystyle{ t\in \mathbb{R}}\):
\(\displaystyle{ f(t) = t^{2} + f(0)}\)
i wystarczy sprawdzić, że dla dowolnego \(\displaystyle{ c\in \mathbb{R}}\) funkcja:
\(\displaystyle{ f(x) = x^{2} + c}\) spełnia zadane równanie.
Awatar użytkownika
mol_ksiazkowy
Użytkownik
Użytkownik
Posty: 11265
Rejestracja: 9 maja 2006, o 12:35
Płeć: Mężczyzna
Lokalizacja: Kraków
Podziękował: 3143 razy
Pomógł: 747 razy

[MIX] Świętokrzyskie warsztaty, wrzesień 2007

Post autor: mol_ksiazkowy »

* 3. dzień, spotkanie popołudniowe:

1. Podać przykład wielomianu, który dla m różnych liczb naturalnych przyjmuje wartości będące różnymi liczbami pierwszymi.

2. Liczby \(\displaystyle{ p}\) i \(\displaystyle{ 8p-1}\) są pierwsze. Wykazać, że liczba \(\displaystyle{ 8p+1}\) jest złożona.

3. Wyznaczyć wszystkie rozwiązania całkowitoliczbowe równania \(\displaystyle{ 3x+7y=1}\)
1 \(\displaystyle{ w(x)= x((x-p_1)...(x-p_m)+1)}\), tj \(\displaystyle{ w(p_j)= p_j}\)

2, musi byc \(\displaystyle{ p>3}\), Jesli \(\displaystyle{ p-1}\) dzieli sie przez \(\displaystyle{ 3}\), to \(\displaystyle{ 8p+1}\) także, a gdy \(\displaystyle{ p=2 \bmod 3}\) to \(\displaystyle{ 8p-1}\) nie jest l. pierwsza


3.
\(\displaystyle{ 3x+7y=1}\)
to
\(\displaystyle{ 3(x+2)+7(y-1)=0}\)
tj .\(\displaystyle{ x+2 = 7t .\ \ t \in\ZZ}\)
a wiec \(\displaystyle{ y=1-3t}\)
Awatar użytkownika
Sylwek
Użytkownik
Użytkownik
Posty: 2716
Rejestracja: 21 maja 2007, o 14:24
Płeć: Mężczyzna
Lokalizacja: Warszawa
Podziękował: 160 razy
Pomógł: 657 razy

[MIX] Świętokrzyskie warsztaty, wrzesień 2007

Post autor: Sylwek »

Chętnie zobaczę rozwiązania następujących zadań (uważam je za najtrudniejsze):
- 2. dzień, spotkanie przedpołudniowe - zadanie 8.
- 2. dzień, spotkanie popołudniowe - zadanie 2.
- 4. dzień, mecz matematyczny - zadanie 2.
- 4. dzień, mecz matematyczny - zadanie 5. (na mocy czego)
- 4. dzień, mecz matematyczny - zadanie 8. (tu jakby ktoś robił to proszę o dowiedzenie, dlaczego ten trójkąt ma pole nie większe niż 0,01 )
- 5. dzień, spotkanie przedpołudniowe - zadanie 2.
- 5. dzień, spotkanie przedpołudniowe - zadanie 6. (dla katów ;P )

Polecam spróbować je rozwiązać

P.S. Czy można prosić o przyklejenie tematu?
Awatar użytkownika
mol_ksiazkowy
Użytkownik
Użytkownik
Posty: 11265
Rejestracja: 9 maja 2006, o 12:35
Płeć: Mężczyzna
Lokalizacja: Kraków
Podziękował: 3143 razy
Pomógł: 747 razy

[MIX] Świętokrzyskie warsztaty, wrzesień 2007

Post autor: mol_ksiazkowy »

Sylwek napisał:
Chętnie zobaczę rozwiązania następujących zadań (uważam je za najtrudniejsze):
- 2. dzień, spotkanie przedpołudniowe - zadanie 8.
\(\displaystyle{ \frac{1 2!}{2}+\frac{2 3!}{2^2}+\frac{3 4!}{2^3}+\ldots+\frac{n (n+1)!}{2^n}= \sum_k \frac{(k+2)! -2(k+1)!}{2^k} = \\ 4(\sum_k \frac{(k+2)!}{2^{k+2}}- \sum_k \frac{(k+1)!}{2^{k+1}} ) = 4(a_3+ ....+a_{n+1}+a_{n+2}- (a_2+....+a_n+a_{n+1}))= 4(a_{n+2}-a_2) = 4(\frac{(n+2)!}{2^{n+2}} -\frac{1}{2})}\).

\(\displaystyle{ a_j =\frac{j!}{2^j}}\)

[ Dodano: 21 Września 2007, 02:02 ]
Sylwek napisał:
- 5. dzień, spotkanie przedpołudniowe - zadanie 2.
Tym razem rzecz formułuje sie klasykiem: Ile jest podzbiorów k -element zbioru n elementowego \(\displaystyle{ \{1, ...,n\}}\), t ze nie zawierają one w sobie żadnych dwóch kolejnych liczb.? \(\displaystyle{ 0 \leq k \leq n}\)- dopuszczamy zbior pusty.... raczej dość łatwo pokazac,,,np indukcja po k , ze jest ich \(\displaystyle{ {n-k+1 \choose k}}\).
W naszym problemie - numerujemy kolejne krzesła: 1, 2.....n i te na którym posadzono osobę są wybrane, tj tworza podzbior k-element, A wiec juz mamy gotowa odpowiedz na nasze zadanie:
\(\displaystyle{ w={n+1 \choose 0}+ {n \choose 1}+ {n-1 \choose 2}+...}\)

I tak np gdy
\(\displaystyle{ n=5\\
w=1+5+6+1=13 }\)

sprawdzamy:

\(\displaystyle{ \emptyset}\)
1


\(\displaystyle{ \{ 1\}, \ \{ 2\}, \ \{ 3\}, \ \{ 4\}, \ \{ 5\}}\)
5


\(\displaystyle{ \{ 1, 3\}, \ \{ 1,4\}, \ \{ 1, 5\}, \ \{ 2, 4\}, \ \{ 2, 5\} , \ \{ 3, 5\}}\)
6


\(\displaystyle{ \ \{ 1, 3, 5\}}\)
1

[ Dodano: 21 Września 2007, 02:23 ]
Sylwek napisaL
- 4. dzień, mecz matematyczny - zadanie 8. (tu jakby ktoś robił to proszę o dowiedzenie, dlaczego ten trójkąt ma pole nie większe niż 0,01 )
Opieramy sie na lemacie: Jesli trojkat zawarty jest we wnetrzu pewnego prostokata, to pole tegoż trójkata nie przekracza połowy pola prostokata, Dowod chyba łatwy.....Potem idzie zas. szufladkowa: kroimy nasz kwadrat na piecdziesiat "plasterków" , tj przystajacych prostokatow, i którys z nich zawierac musi trzy punkty -i dalej to nasz lemat wykancza sprawe,
Awatar użytkownika
max
Użytkownik
Użytkownik
Posty: 3306
Rejestracja: 10 gru 2005, o 17:48
Płeć: Mężczyzna
Lokalizacja: Lebendigentanz
Podziękował: 37 razy
Pomógł: 778 razy

[MIX] Świętokrzyskie warsztaty, wrzesień 2007

Post autor: max »

Na 5 dzień przedpołudniowe 2. można spojrzeć też nieco inaczej, mianowicie niech \(\displaystyle{ d_{n}}\) oznacza liczbę sposobów dla \(\displaystyle{ n}\) krzeseł, nietrudno pokazać, że:
\(\displaystyle{ \begin{cases}d_{1} = 2\\
d_{2} = 3\\
d_{n + 2} = d_{n + 1} + d_{n}\end{cases}}\)

(w tym ostatnim przypadku zostawiamy ostatnie krzesło puste, lub przedostatnie krzesło puste, a na ostatnim sadzamy osobę)
stąd \(\displaystyle{ d_{n} = f_{n + 2} = \frac{(1 + \sqrt{5})^{n + 2} - (1 - \sqrt{5})^{n + 2}}{2^{n + 2}\sqrt{5}}}\)


Co do 2. z meczu, to implikacja w prawo wynika z wzoru na sumę szeregu geometrycznego, a implikacja w lewo - np z algorytmu dzielenia pisemnego, tzn, ograniczając się do przypadku gdy \(\displaystyle{ \frac{p}{q}}\) jest dodatnim ułamkiem właściwym definiujemy sobie dwa ciągi \(\displaystyle{ (p_{n}), \ (r_{n})}\) jako spełniające warunki:
\(\displaystyle{ r_{1} = p\\
10r_{n} = p_{n}\cdot q + r_{n + 1}, \ \ p_{n}, r_{n + 1} \mathbb{N},\ 0 \leqslant r_{n + 1} < q\\}\)

(w myśl twierdzenia o dzieleniu i zasady indukcji ciągi te są określone jednoznacznie)
stąd oczywiście \(\displaystyle{ p_{n}\in \{0, 1, 2, 3, 4, 5, 6, 7, 8, 9\}}\) i nietrudno widzieć, że ponieważ wyrazy ciągu \(\displaystyle{ (r_{n})}\) należą do skończonego zbioru, to ciąg ten musi być (na mocy jego definicji i tw o dzieleniu) od pewnego miejsca okresowy, a wraz z nim również ciąg \(\displaystyle{ (p_{n})}\). Pozostaje pokazać, że \(\displaystyle{ \sum_{n = 1}^{\infty}\frac{p_{n}}{10^{n}} = \frac{p}{q}}\), bo np \(\displaystyle{ 0 \leqslant \frac{p}{q} - \sum_{k = 1}^{n}\frac{p_{k}}{10^{k}} < \frac{1}{10^{n}}}\)

Powyższe rozumowanie łatwo uogólnia się na przypadki systemów pozycyjnych o dowolnych podstawach naturalnych większych niż \(\displaystyle{ 1}\).
Ostatnio zmieniony 21 wrz 2007, o 23:50 przez max, łącznie zmieniany 1 raz.
Piotr Rutkowski
Użytkownik
Użytkownik
Posty: 2234
Rejestracja: 26 paź 2006, o 18:08
Płeć: Mężczyzna
Lokalizacja: Warszawa
Podziękował: 22 razy
Pomógł: 390 razy

[MIX] Świętokrzyskie warsztaty, wrzesień 2007

Post autor: Piotr Rutkowski »

Hmm, mól książkowy, czy mógłbyś zaprezentować dowód tego lematu o prostokącie. Bo ja rozwiązując zadanie Sylwka doszedłem to takich samych wniosków co ty, ale właśnie problem miałem z dowodem o trójkącie w prostokącie. (chyba miałem nieodpowiednie podejście)
Awatar użytkownika
mol_ksiazkowy
Użytkownik
Użytkownik
Posty: 11265
Rejestracja: 9 maja 2006, o 12:35
Płeć: Mężczyzna
Lokalizacja: Kraków
Podziękował: 3143 razy
Pomógł: 747 razy

[MIX] Świętokrzyskie warsztaty, wrzesień 2007

Post autor: mol_ksiazkowy »

hm, nop rozwiazac to mozna róznie, jak mysle , ale np tak: szkic, jest trojkat ABC w prostokacie PQRS i najblizej boku PS jest wierzch. A , zas QR jest wierzch. C. i wtedy \(\displaystyle{ A^\prime, B^\prime, C^\prime}\) to rzuty punktow A, B, C na bok PQ. D to punkt na AC t. ze BD jest rownolegle do PS, i:
Pole (trojkata ABC) =\(\displaystyle{ \frac{1}{2}A^\prime C^\prime BD}\) i \(\displaystyle{ A^\prime C^\prime \leq PQ, \ \ BD \leq QR}\)
A Twoja metoda jaka...?
Ostatnio zmieniony 21 wrz 2007, o 18:32 przez mol_ksiazkowy, łącznie zmieniany 2 razy.
Piotr Rutkowski
Użytkownik
Użytkownik
Posty: 2234
Rejestracja: 26 paź 2006, o 18:08
Płeć: Mężczyzna
Lokalizacja: Warszawa
Podziękował: 22 razy
Pomógł: 390 razy

[MIX] Świętokrzyskie warsztaty, wrzesień 2007

Post autor: Piotr Rutkowski »

Moja była taka sama jak Twoja (Dirichlet, choć ociupinkę inaczej (podzieliłem kwadrat na 100 małych kw. , istnieje taki mały kwadrat, w którym są 2 punkty, w którymś z sąsiednich musi być jakiś punkt, bo jak nie to... itd.) czyli praktycznie to samo) natomiast u mnie brakowało właśnie dowodu tego lematu. Rozmawiałem trochę z Sylwkiem i też mu to mówiłem. U niego na obozie robili jakieś cuda niewidy żeby to udowodnić. Zakładali, że jeden wierzchołek trójkąta jest w wierzchołku prostokąta, a jeśli nie jest to sobie "ucinali" ten prostokąt, ale podobno za ich metodę i tak ucięli punkty.
Awatar użytkownika
mol_ksiazkowy
Użytkownik
Użytkownik
Posty: 11265
Rejestracja: 9 maja 2006, o 12:35
Płeć: Mężczyzna
Lokalizacja: Kraków
Podziękował: 3143 razy
Pomógł: 747 razy

[MIX] Świętokrzyskie warsztaty, wrzesień 2007

Post autor: mol_ksiazkowy »

Sylwek napisał
2. Odcinki AB i CD są styczne do dwóch okręgów, do pierwszego w punktach A i C, do drugiego w punktach B i D. Udowodnić, że rzuty odcinków AC i BD na prostą łączącą środki tych okręgów są równej długości.
aha, a czy tu nie chodzi moze o rzuty odc AD i Bc, bo chyba w tej wersji sie to
upraszcza i wszystko zapada do punktu...czy nie ?
sorki bo moze rys mam cos nie tak..
, etc
Awatar użytkownika
Tristan
Użytkownik
Użytkownik
Posty: 2353
Rejestracja: 24 kwie 2005, o 14:28
Płeć: Mężczyzna
Podziękował: 27 razy
Pomógł: 557 razy

[MIX] Świętokrzyskie warsztaty, wrzesień 2007

Post autor: Tristan »

Ad 1.pe.1:
Skorzystamy tu z jednego z wzorów skróconego mnożenia. Po podanym przykładzie łatwo rozstrzygnąć przypadek ogólny.
\(\displaystyle{ \frac{ x^8 +x^6 +x^4 +x^2 +1}{ x^4 +x^3 +x^2 +x+1}=\frac{ (x^2)^4 +(x^2)^3 +(x^2)^2 + (x^2)^1 +1}{ x^4 +x^3 +x^2 +x+1}= \frac{ (x^2)^4 +(x^2)^3 +(x^2)^2 + (x^2)^1 +1}{ x^4 +x^3 +x^2 +x+1} \frac{ x^2 -1}{ (x-1)(x+1) }= \frac{ (x^2)^5 - 1}{ (x^5 - 1)(x+1) }= \frac{ (x^5 -1)( x^5 +1) }{ (x^5 - 1)(x+1) }= \frac{ x^5 +1}{ x+1}}\)
Ad 1.pe.6:
Zauważmy, że \(\displaystyle{ x^3 - 3= x^3 - 27 +24=(x-3)(x^2 +3x+9)+24}\). Zachodzi więc implikacja \(\displaystyle{ x-3|x^3 - 3 x-3|24}\). Oznacza to, że:
\(\displaystyle{ x-3=24 x-3=12 x-3=8 x-3=6 x-3=4 x-3=3 \\ x-3=2 x-3=1 x-3=-1 x-3=-2 x-3=-3 \\ x-3=-4 x-3=-6 x-3=-8 x-3=-12 x-3=-24}\)
Ostatecznie \(\displaystyle{ x \{ -21,-9,-5,-3,-1,0,1,2,4,5,6,7,9,11,15,27 \}}\)
ODPOWIEDZ